NURSE 405 Quiz 3

Pataasin ang iyong marka sa homework at exams ngayon gamit ang Quizwiz!

The emergency department nurse is assessing a child who has presented with a 2-day history of nausea and vomiting with pain that is isolated to the right upper quadrant of the abdomen. Which action is most appropriate? Encourage fluid intake. Assess the child's usual urinary voiding pattern. Administer antacids as ordered. Prepare the child for admission to the hospital.

Prepare the child for admission to the hospital. The child's presentation is consistent with cholecystitis, which necessitates surgery in most cases. The child should be kept NPO and antacids are of no benefit. Genitourinary involvement is atypical.

A client who was injured while playing basketball reports an extremely painful elbow, which is very edematous. What type of injury has the client experienced? sprain All options are correct. contusion strain

sprain Explanation: Sprains are injuries to the ligaments surrounding a joint. A strain is an injury to a muscle when it is stretched or pulled beyond its capacity. A contusion is a soft tissue injury resulting from a blow or blunt trauma. Sprains are injuries to the ligaments surrounding a joint.

A client in the emergency department is being treated for a wrist fracture. The client asks why a splint is being applied instead of a cast. What is the best response by the nurse? "It is best if an orthopedic doctor applies the cast." "A splint is applied when more swelling is expected at the site of injury." "You would have to stay here much longer because it takes a cast longer to dry." "Not all fractures require a cast."

"A splint is applied when more swelling is expected at the site of injury." Explanation: Splints are noncircumferential and will not compromise circulation when swelling is expected. A splint is applied to support and immobilize the injured joint. A fracture will swell as part of the inflammation process. The client would not have to stay longer if a fiberglass cast is applied. Fiberglass cast dry in approximately 30 minutes. An orthopedic doctor is not needed to apply the cast. Many nurses and technicians are trained in proper application of a cast. Some fractures may not be treated with a cast but it would not be appropriate to answer with this response because it does not reflect the actual reason for a splint being applied.

Following a total knee replacement, the surgeon orders a continuous passive motion (CPM) device. The client asks about the purpose of this treatment. What is the best response by the nurse? "CPM increases range of motion of the joint." "CPM prevents injury by limiting flexion of the knee." "CPM delivers analgesic agents directly into the joint." "CPM strengthens the muscles of the leg."

"CPM increases range of motion of the joint." Explanation: CPM increases circulation and range of motion of the knee joint.

A client with a recent left above-the-knee amputation states, "I can feel pain in my left toes." Which is the best response by the nurse? "Describe the pain and rate it on the pain scale." "The pain is really from the nerves in the upper leg." "Pain medication usually does not help this type of pain." "Your left toes have been amputated."

"Describe the pain and rate it on the pain scale." Explanation: The nurse should recognize phantom pain as real and complete a pain assessment as if the limb were attached. The client's pain should be address and treated appropriately. By telling the client that the toes have been amputated or the pain is really from the nerves in the upper leg, the nurse is negating the client's pain. Opioid pain medication can be effective with phantom pain.

A 15-year-old adolescent is scheduled for a pelvic ultrasound to evaluate for a possible ovarian cyst. Which instruction by the nurse would be most appropriate? "Limit your level of physical activity for one-half hour before the test." "You need to remain very still for the entire test." "You won't be able to drink any water before or during the test." "Drink plenty of fluids because you need to have a full bladder."

"Drink plenty of fluids because you need to have a full bladder." A full bladder is needed for an ultrasound of the pelvic region. The client needs to remain still for a computed tomography or magnetic resonance imaging scan, not an ultrasound. Water is withheld during a water deprivation test used to detect diabetes insipidus. Limiting stress and physical activity for 30 minutes before the test is required for the growth hormone stimulation test.

The nurse is caring for an infant recently diagnosed with oral candidiasis (thrush) who has been prescribed nystatin. Which statement by the infant's mother would suggest a need for further education? "I will use a cotton tipped applicator to apply the medication to her mouth." "I will make sure to clean all of her toys before I give them to her." "I will add the nystatin to her bottle four times per day." "I will watch for diaper rash."

"I will add the nystatin to her bottle four times per day." Administer nystatin suspension four times per day following feeding, not mixed in the bottle, to allow the medication to remain in contact with the lesions

The nurse is teaching glucose monitoring and insulin administration to a child with type 1 diabetes and the parents. Which comment by a parent demonstrates a need for additional teaching? "We should check our child's blood glucose levels before meals." "If our child is sick we should check blood glucose levels more often." "During exercise we should wait to check blood sugars until after our child completes the activity." "Blood glucose level, food intake, and activity need to all be considered when calculating insulin dosage."

"During exercise we should wait to check blood sugars until after our child completes the activity." Blood glucose monitoring needs to be performed more often during prolonged exercise. Frequent glucose monitoring before, during, and after exercise is important to recognize hypoglycemia or hyperglycemia.

The nurse is assessing a toddler and palpates a sausage-shaped mass in the upper mid abdomen. When taking the toddler's history, what question would the nurse ask the parent first? "Has your toddler been having different colored stools?" "Has your toddler been around anyone who has been sick?" "How is your toddler's appetite?" "Can you describe any pain your toddler is having?"

"Has your toddler been having different colored stools?" A sausage-shaped mass in the upper mid abdomen is a classic sign of intussusception. Intussusception occurs when the proximal segment of the bowel "telescopes" into a more distal segment of the bowel, thus the sausage-shaped mass. Another classic sign of intussusception is stools that appear like currant jelly. These are stools which are bloody and mixed with mucus. This should be the question the nurse asks first. Next, the nurse should ask about the pain. The pain with intussusception has a sudden onset and is intermittent and crampy. The appetite of the child generally is poor due to abdominal pressure and pain. Intussusception is not a contagious or infectious disease.

During a visit to the clinic, the adolescent client with hypothyroidism tells the nurse that she takes her levothyroxine "whenever I think about it...sometimes I miss a dose, but not very often." What is the best response by the nurse?

"I know it's hard to remember medicines, but it is really important for you to take it before breakfast each day to control your hypothyroidism." Levothyroxine is a thyroid hormone replacement used to treat hypothyroidism. It is important to maintain a consistent thyroid hormone level by taking the medication at the same time each day (preferably 30 minutes prior to breakfast for best absorption). Toxicity can occur if the dose is doubled. Suggesting the client "do something" to remember does not highlight the importance of taking it correctly.

The nurse is providing instructions to the client who is being prepared for skeletal traction. Which statement by the client indicates teaching was effective? "I will wear a boot with weights attached." "Metal pins will go through my skin to the bone." "A belt will go around my pelvis and weights will be attached." "The traction can be removed once a day so I can shower."

"Metal pins will go through my skin to the bone." Explanation: In skeletal traction, metal rods or pins are used to apply continuous traction directly to the bone. Weights are used to apply the traction. Casts, external fixators, or splints are used when the traction is discontinued.

A nurse is caring for a 6-year-old girl recently diagnosed with celiac disease and is discussing dietary restrictions with the girl's mother. Which response indicates a need for further teaching? "My daughter is eating more vegetables." "My daughter can eat any kind of fruit." "There are many types of flour besides wheat." "There is gluten hidden in unexpected foods."

"My daughter can eat any kind of fruit." While most fruits and fruit juices are allowed, the nurse needs to make sure the mother knows that some fruit pie fillings and dried fruit may contain gluten.

A 13-year-old adolescent with hyperthyroidism who takes antithyroid medication has a sore throat and a fever. The parent calls the nurse and asks what to do. Which is the best response from the nurse? "Fever and sore throat may be side effects of the medication." "Please take your child straight to the emergency department." "Offer your child at least 8 ounces of clear fluids and call back tomorrow." "Give your child ibuprofen according to the instructions on the box."

"Please take your child straight to the emergency department." A side effect of antithyroid medications is leukopenia. Signs and symptoms that include fever and sore throat need to be seen immediately. These instructions should be reviewed with parents upon discharge. The question includes information about Graves disease, so ibuprofen would not be the treatment. The question centers around drug therapy, not the child's fluid status.

The nurse is caring for an infant. The infant's mother asks the nurse, "What did the doctor mean when he said she may have regurgitation?" What response by the nurse is appropriate? "Regurgitation is not normal in infants. She will need more testing to see what is causing this." "Regurgitation is when an infant can't tolerate their formula. You will need to switch." "Regurgitation is just another term for vomiting. All infants vomit some." "Regurgitation is the backflow of stomach contents up into the esophagus or mouth."

"Regurgitation is the backflow of stomach contents up into the esophagus or mouth." Regurgitation is the backflow of stomach contents up into the esophagus and/or oral cavity. The muscle tone of the lower esophageal sphincter is not fully developed until age 1 month, so infants younger than 1 month of age frequently regurgitate after feedings. Many children younger than 1 year of age continue to regurgitate for several months, but this usually disappears with age.

After explaining the causes of hypothyroidism to the parents of a newly diagnosed infant, the nurse should recognize that further education is needed when the parents ask which question? "Do you mean that hypothyroidism may be caused by a problem in the way the body makes thyroxine?" "Are you saying that hypothyroidism is caused by a problem in the way the thyroid gland develops?" "So, hypothyroidism can be only temporary, right?" "So, hypothyroidism can be treated by exposing our baby to a special light, right?"

"So, hypothyroidism can be treated by exposing our baby to a special light, right?" Congenital hypothyroidism can be permanent or transient and may result from a defective thyroid gland or an enzymatic defect in thyroxine synthesis. Only the last question, which refers to phototherapy for physiologic jaundice, indicates that the parents need more information.

The student nurse is preparing a presentation on celiac disease. What information should be included? Select all apply. "Symptoms of celiac disease include diarrhea, steatorrhea, anemia, and dental disorders." "Most children with celiac disease are diagnosed within the first year of life." "The only treatment for celiac disease is a strict gluten-free diet." "Gluten is found in most wheat products, rye, barley and possibly oats." "The entire family will need to eat a gluten-free diet."

"Symptoms of celiac disease include diarrhea, steatorrhea, anemia, and dental disorders." "The only treatment for celiac disease is a strict gluten-free diet." "Gluten is found in most wheat products, rye, barley and possibly oats."

The nurse is assessing a child diagnosed with Cushing syndrome. Which statement by the parents demonstrates a need for further teaching? "My child's round, full face appearance is reversible with appropriate treatment." "My child may experience excessive weight gain." "This disorder is most likely due to an infection my child had recently." "We need to pay close attention to any wounds our child gets to monitor for adequate healing."

"This disorder is most likely due to an infection my child had recently." A round, full face (moon face), rapid weight gain, and poor wound healing are all seen in Cushing syndrome. Cushingoid appearance is reversible with appropriate treatment. The most common cause of Cushing syndrome is long-term corticosteroid therapy or a pituitary adenoma, not an infection.

The nurse is preparing a client for surgery and the client asks, "What does it mean when they say they are going to reduce my abdominal hernia?" Which response by the nurse is most appropriate? "This means the hernia contents will be manipulated back into your peritoneal cavity during surgery." "All this means is that the herniated intestines are twisted and edematous, which is why you need surgery." "The health care provider will remove about half of the herniated contents during the procedure." "If you do not understand this, I need to cancel your surgery and have the health care provider come back."

"This means the hernia contents will be manipulated back into your peritoneal cavity during surgery." A hernia in the abdominal region is considered reducible when its contents are easily manipulated back through the inguinal ring into the peritoneal cavity. The nurse would reinforce this education, already provided by the primary health care provider when the surgery was explained, to the client. It is not necessary to cancel surgery when the nurse can provide education to the client. Reducing does not mean the intestines are twisted and edematous. Nor does it mean half of the contents will be removed.

The nurse is speaking with the parents of a child recently diagnosed with hypothyroidism. Which statement by a parent indicates an understanding of symptoms of this disorder? "My son's nervousness may be a symptom of his hypothyroidism." "Heat intolerance is a caused by low thyroid levels." "When they get my son's thyroid levels normal, he won't be so tired." "Most people with hypothyroidism have smooth, velvety skin."

"When they get my son's thyroid levels normal, he won't be so tired." Tiredness, fatigue, constipation, cold intolerance and weight gain are all symptoms of hypothyroidism. Nervousness, anxiety, heat intolerance, weight loss and smooth velvety skin are all symptoms of hyperthyroidism

A student accepted into a nursing program must begin receiving the hepatitis B series of injections. The student asks when the next two injections should be administered. What is the best response by the instructor? "You must have the second one in 2 weeks and the third in 1 month." "You must have the second one in 1 year and the third the following year." "You must have the second one in 1 month and the third in 6 months." "You must have the second one in 6 months and the third in 1 year."

"You must have the second one in 1 month and the third in 6 months." Both forms of the hepatitis B vaccine are administered intramuscularly in three doses; the second and third doses are given 1 and 6 months, respectively, after the first dose.

A 10-year-old client is upset and tells the school nurse. "I am the shortest one in my class. I am done with school and just want to stay home." Which response by the nurse is most appropriate? "You seem very upset. Sit down and let's talk about what is going on." "I know kids can be mean these days, but I am sure you will be taller soon." "Let me explain to you your disorder so you understand what is going on." "Would you like to talk with another child who has experienced what you are going through?"

"You seem very upset. Sit down and let's talk about what is going on."

The nurse is preparing to administer the child's ordered lispro (Humalog) insulin at 0800. When will the child's blood glucose level begin to decline? 0845 0815 0900 0930

0815

A newborn is born with hypothyroidism. If it is not recognized and treated, what complication is likely? muscle spasticity blindness dehydration cognitive impairment

220 mg/dl A fasting blood sugar result of 200 mg/dL or more almost certainly is diagnostic for diabetes when other signs, such as polyuria and weight loss despite polyphagia, are present.

When collecting data on a child diagnosed with diabetes mellitus, the nurse notes that the child has had weight loss and other symptoms of the disease. The nurse would anticipate which finding in the child's fasting glucose levels? 60 mg/dl 220 mg/dl 140 mg/dl 100 mg/dl

220 mg/dl A fasting blood sugar result of 200 mg/dL or more almost certainly is diagnostic for diabetes when other signs, such as polyuria and weight loss despite polyphagia, are present.

An older adult patient had a hip replacement. When should the patient begin with assisted ambulation with a walker? 2 to 3 weeks 24 hours 1 week 72 hours

24 hours Explanation: Following hip arthroplasty (total hip replacement), patients begin ambulation with the assistance of a walker or crutches within a day after surgery.

A client is hospitalized for open reduction of a fractured femur. During the postoperative assessment, the nurse notes that the client is restless and observes petechiae on the client's chest. Which nursing action is indicated first? Administer oxygen. Contact the nursing supervisor. Elevate the affected extremity. Contact the health care provider.

Administer oxygen. Explanation: The client is demonstrating clinical manifestations consistent with a fatty embolus. Administering oxygen is the top priority. Elevating the extremity won't alter the client's condition. Notifying the nursing supervisor may be indicated by facility policy after other immediate actions have been taken. The nurse should contact the health care provider after administering oxygen.

An older adult client slipped on an area rug at home and fractured the left hip. The client is unable to have surgery immediately and is having severe pain. What interventions should the nurse provide for the patient to minimize energy loss in response to pain? Administer prescribed analgesics around-the-clock. Give pain medication to the client after providing care. Avoid administering too much medication because the client is older. Administer prescribed pain medication only when the client requests it.

Administer prescribed analgesics around-the-clock. Explanation: Pain associated with hip fracture is severe and must be carefully managed with around-the-clock dosing of pain medication to minimize energy loss in response to pain. The client may not request the medication even if they are in pain, and it should be offered at the prescribed time. Give pain medication prior to providing any type of care involved in moving the client.

The school nurse notes that a child diagnosed with diabetes mellitus is experiencing an insulin reaction and is unable to eat or drink. Which action would be the most appropriate for the school nurse to take? Dissolve a piece of candy in the child's mouth. Anticipate that the child will need intravenous glucose. Request that someone call 911. Administer subcutaneous glucagon.

Administer subcutaneous glucagon. If the child having an insulin reaction cannot take a sugar source orally, glucagon should be administered subcutaneously to bring about a prompt increase in the blood glucose level. This treatment prevents the long delay while waiting for a physician to administer IV glucose or for an ambulance to reach the child.

Prior to discharging an infant with congenital hypothyroidism to home with the parents, what should the nurse emphasize regarding the care that this child will need going forward? Vitamin K administration until school age Administration of levothyroxine indefinitely An increased intake of calcium beginning immediately Administration of vitamin C until after growth is complete

Administration of levothyroxine indefinitely The treatment for hypothyroidism is oral administration of synthetic thyroid hormone or sodium levothyroxine. A small dose is given at first, and then the dose is gradually increased to therapeutic levels. The child needs to continue taking the synthetic thyroid hormone indefinitely to supplement that which the thyroid does not make. Vitamin K is not needed. Supplemental vitamin D, and not calcium, may be given to prevent the development of rickets when rapid bone growth begins. Supplemental vitamin C is not indicated for this disorder.

A client with liver and renal failure has severe ascites. On initial shift rounds, his primary nurse finds his indwelling urinary catheter collection bag too full to store more urine. The nurse empties more than 2,000 ml from the collection bag. One hour later, she finds the collection bag full again. The nurse notifies the physician, who suspects that a bladder rupture is allowing the drainage of peritoneal fluid. The physician orders a urinalysis to be obtained immediately. The presence of which substance is considered abnormal? Urobilinogen Albumin Creatinine Chloride

Albumin Explanation: Albumin is an abnormal finding in a routine urine specimen. Ascites present in liver failure contain albumin; therefore, if the bladder ruptured, ascites containing albumin would drain from the indwelling urinary catheter because the catheter is no longer contained in the bladder. Creatinine, urobilinogen, and chloride are normally found in urine.

A client has been involved in a motor vehicle collision. Radiographs indicate a fractured humerus; the client is awaiting the casting of the upper extremity and admission to the orthopedic unit. Other than the bone, what physical structures could be affected by this injury? nerves blood vessels muscles All options are correct.

All options are correct. Explanation: A fractured bone or other injury can potentially cause dysfunction to the surrounding muscle and injury to the blood vessels and nerves.

The single modality of pharmacologic therapy for chronic type B viral hepatitis is: Hepsera Epivir Baraclude Alpha-interferon

Alpha-interferon Explanation: Alpha-interferon is a biologic response modifier that is highly effective for treatment of hepatitis B. The other antiviral agents are effective but not the preferred single-agent therapy.

A nurse is caring for a client who is in skeletal traction. To prevent the complication of skin breakdown in a client with skeletal traction, what action should be included in the plan of care? Encourage the client to perform isometric exercises once a shift. Assess the pin insertion site every 8 hours. Encourage the client to push up with the elbows when repositioning. Apply occlusive dressings to the pin sites.

Assess the pin insertion site every 8 hours. Explanation: The pin insertion site should be assessed every 8 hours for inflammation and infection. Loose cover dressings should be applied to pin sites. The client should be encouraged to use the overhead trapeze to shift weight for repositioning. Isometric exercises should be done 10 times an hour while awake.

A nurse is caring for a client placed in traction to treat a fractured femur. Which nursing intervention has the highest priority? Keeping the client from sliding to the foot of the bed Keeping the ropes over the center of the pulley Assessing the extremity for neurovascular integrity Ensuring that the weights hang free at all times

Assessing the extremity for neurovascular integrity Explanation: Although all measures are correct, assessing neurovascular integrity takes priority because a decrease in neurovascular integrity could compromise the limb. The pull of the traction must be continuous to keep the client from sliding. Sufficient countertraction must be maintained at all times by keeping the ropes over the center of the pulley. The line of pull is maintained by allowing the weights to hang free.

A group of students are reviewing information about cast composition in preparation for a discussion on the advantages and disadvantages of each. The students demonstrate understanding of the topic when they cite which of the following as an advantage of a plaster cast? Quicker drying Longer-lasting More breathable Better molding to the client

Better molding to the client Explanation: Plaster casts require a longer time for drying, but mold better to the client, and are initially used until the swelling subsides. Fiberglass casts dry more quickly, are lighter in weight, longer-lasting, and breathable.

A 6-year-old child is being evaluated for growth hormone dysfunction. Which tests will be employed in the diagnostic workup? Select all that apply. MRI erythrocyte sedimentation rate pituitary function test complete blood count CT scan

CT scan MRI pituitary function test The child will undergo laboratory tests to rule out chronic illnesses such as renal failure or liver and thyroid dysfunction. Laboratory and diagnostic tests used in children with suspected GH deficiency include CT and MRI to assess for structural abnormalities. A pituitary function test will be used to confirm a diagnosis of growth hormone dysfunction.

A newborn exhibits significant jittery movements, convulsions, and apnea. Hypoparathyroidism is suspected. What would the nurse expect to be administered? Calcium gluconate Desmopressin Levothyroxine Hydrocortisone

Calcium gluconate Intravenous calcium gluconate is used to treat acute or severe tetany. Hydrocortisone is used to treat congenital adrenal hyperplasia and Addison disease. Desmopressin is used to control diabetes insipidus. Levothyroxine is a thyroid hormone replacement used to treat hypothyroidism.

A nurse is caring for a client in skeletal leg traction. Which nursing assessment findings indicate the client has met expected outcomes? Select all that apply. Peripheral pulses +2 bilaterally Repositions self with trapeze Right calf warm and swollen Elbows are free of skin breakdown Capillary refill less than 3 seconds

Capillary refill less than 3 seconds Repositions self with trapeze Peripheral pulses +2 bilaterally Elbows are free of skin breakdown

An important nursing assessment, post fracture, is to evaluate neurovascular status. Therefore, the nurse should check for: Crepitus. Shortening and deformity. Capillary refill. Swelling and discoloration.

Capillary refill. Explanation: Assessment for neurovascular impairment includes checking for weak pulses or delayed capillary refill (normal is <2 seconds).

The nurse is administering an enteral feeding to a child with a gastrostomy tube (G-tube). Which action will the nurse take when administering a prescribed feeding through the client's G-tube? After feeding, flush the tube with a small amount of saline and leave the G-tube open for 2 minutes. Use a syringe plunger to administer the feeding. Check for gastric residual before starting feeding. Position the client with the head of the bed at a 20° angle.

Check for gastric residual before starting feeding. The nurse should check for gastric residual before starting feeding by gently aspirating from the tube with a syringe or positioning the tube below the level of the stomach with only the barrel of the syringe attached. The client should be positioned with his or her head elevated 30° to 45° and the formula should be allowed to flow with gravity, not plunged unless the tube is clogged. After feeding, the nurse should flush the tube with a small amount of water, unless contraindicated, and leave the G-tube open for 5 to 10 minutes after feeding to allow for escape of air.

The nurse is caring for a client who was involved in an automobile accident and sustained multiple trauma. The client has a Volkmann's contracture to the right hand. What objective data does the nurse document related to this finding? Claw-like deformity of the right hand without ability to extend fingers Extension of the fingers of the right hand Dislocation of the fingers Nodules on the knuckles of the third and fourth finger

Claw-like deformity of the right hand without ability to extend fingers Explanation: A Volkmann's contracture is a claw like deformity of the hand resulting from obstructed arterial blood flow to the forearm and hand. The client is unable to extend the fingers and complains of unrelenting pain, particularly if attempting to stretch the hand. Nodule on the knuckles and dislocation are not indicative of Volkmann's contracture.

Which of the following type of fracture is associated with osteoporosis? Simple Oblique Stress Compression

Compression Explanation: Compression fractures are caused by compression of vertebrae and are associated frequently with osteoporosis. Stress fractures occur with repeated bone trauma from athletic activities, most frequently involving the tibia and metatarsals. An oblique fracture runs across the bone at a diagonal angle of 45 to 60 degrees. A simple fracture (closed fracture) is one that does not cause a break in the skin.

A nurse working in a pediatric clinic is examining a child with symptoms indicating a possible inborn error of metabolism. Which action is most important for the nurse to do at this time? Ignore the symptoms if the family does not ask about them. Counsel the family to have all siblings evaluated. Read more about the condition and its ramifications. Reassure the family that everything is OK, but express urgency to the health care provider.

Counsel the family to have all siblings evaluated. If one child in the family has suspicious symptoms, counsel the family to have all other siblings evaluated, even if their symptoms are not exactly the same. The nurse should never ignore such a problem and should never tell a family not to worry when a problem may exist. Reading more about the problem may be helpful, but it is not a priority action at this time.

Which would be contraindicated as a component of self-care activities for the client with a cast? Cover the cast with plastic to insulate it Do not attempt to scratch the skin under a cast Cushioning rough edges of the cast with tape Elevate the casted extremity to heart level frequently

Cover the cast with plastic to insulate it Explanation: The cast should be kept dry, but do not cover it with plastic or rubber because this causes condensation, which dampens the cast and skin. The other activities are consistent with cast care.

A child presents with intermittent abdominal pain, severe anorexia, and diarrhea. The child's height and weight are significantly behind standards for age. There is skin breakdown in the anal region. The nurse explains that this presentation is consistent with which diagnosis? Hirschsprung disease Crohn disease ulcerative colitis food poisoning

Crohn disease Intermittent abdominal pain, anorexia, diarrhea, growth delays, and perianal lesions are characteristic of Crohn disease. In ulcerative colitis, the pain is continuous with bloody diarrhea, but anorexia, weight loss, and growth delay are mild. Food poisoning is an acute condition and may result in weight loss but not growth delays. In Hirschsprung disease the bowel lacks nerve innervation, so it lacks motility and fecal output.

A client with a fractured distal left radius reports discomfort at the cast site, with pain specifically in the upper forearm. What would the nurse expect the physician to do? Cut a cast window. Initiate physical therapy. Remove the cast. Apply a fiberglass cast.

Cut a cast window. Explanation: After the cast dries, a cast window, or opening, may be cut. This usually is done when the client reports discomfort under the cast or has a wound that requires a dressing change. The window permits direct inspection of the skin, a means to check the pulse in a casted arm or leg, or a way to change a dressing.

Which is an age-related change of the hepatobiliary system? Enlarged liver Decreased prevalence of gallstones Decreased blood flow Increased drug clearance capability

Decreased blood flow Age-related changes of the hepatobiliary system include decreased blood flow, decreased drug clearance capability, increased presence of gall stones, and a steady decrease in the size and weight of the liver.

A woman in her first trimester of pregnancy has just been diagnosed with acquired hypothyroidism. The nurse is alarmed because this condition can lead to which pregnancy complication? Congenital heart defects in the fetus Decreased cognitive development of the fetus Spina bifida in the fetus Gestational diabetes in the mother

Decreased cognitive development of the fetus If acquired hypothyroidism exists in a woman during pregnancy, her infant can be born intellectually disabled, because there was not enough iodine present for fetal growth. It is important, therefore, that girls with this syndrome be identified before they reach childbearing age.

A newborn is diagnosed with the salt-losing form of congenital adrenogenital hyperplasia. On what should the nurse focus when assessing this client? Excessive cortisone secretion Bleeding tendency Dehydration Hypoglycemia

Dehydration If there is a complete blockage of cortisol formation, aldosterone production will also be deficient. Without adequate aldosterone, salt is not retained by the body, so fluid is not retained. Almost immediately after birth, affected infants begin to have vomiting, diarrhea, anorexia, loss of weight, and extreme dehydration. If these symptoms remain untreated, the extreme loss of salt and fluid can lead to collapse and death as early as 48 to 72 hours after birth. The salt-losing form must be detected before an infant reaches an irreversible point of salt depletion. T

The parents of a child who was diagnosed with diabetes insipidus ask the nurse, "How does this disorder occur?" When responding to the parents, the nurse integrates knowledge that a deficiency of which hormone is involved? Thyroxine Antidiuretic hormone Growth hormone Insulin

Diabetes insipidus results from a deficiency in the secretion of antidiuretic hormone (ADH). This hormone, also known as vasopressin, is produced in the hypothalamus and stored in the pituitary gland.

Which of the following would lead a nurse to suspect that a client has a rotator cuff tear? Pain worse in the morning Difficulty lying on affected side Minimal pain with movement Increased ability to stretch arm over the head

Difficulty lying on affected side Explanation: Clients with a rotator cuff tear experience pain with movement and limited mobility of the shoulder and arm. They especially have difficulty with activities that involve stretching their arm above their head. Many clients find that the pain is worse at night and that they are unable to sleep on the affected side.

The nurse is caring for a 7-year-old girl diagnosed with precocious puberty. The child is tearful when talking with the nurse about the signs and symptoms of the disorder. She states, "I don't look like my friends." When preparing the care plan for this child, which nursing diagnosis has the highest priority? Imbalanced nutrition Disturbed body image Interrupted family process Deficient knowledge

Disturbed body image In precocious puberty, the child develops sexual characteristics before the usual age of pubertal onset. Disturbed body image would be the highest priority nursing diagnosis based on the child being tearful and the statement about not looking like her friends. Deficient knowledge about the disorder or treatment may apply, but is not the priority in this situation.

A nurse is caring for a patient with cancer of the liver whose condition has required the insertion of a percutaneous biliary drainage system. The nurse's most recent assessment reveals the presence of dark green fluid in the collection container. What is the nurse's best response to this assessment finding? Document the presence of normal bile output. Promptly report this assessment finding to the primary provider. Irrigate the drainage system with normal saline as prescribed. Aspirate a sample of the drainage for culture.

Document the presence of normal bile output. Explanation: Bile is usually a dark green or brownish-yellow color, so this would constitute an expected assessment finding, with no other action necessary.

The nurse is caring for a newborn with 21-OH enzyme deficiency congenital adrenal hyperplasia (CAH). The nurse identifies one goal of the plan of care as being the understanding of the importance of maintaining hormone supplementation. Which outcome criteron demonstrates this goal has been met? During follow-up visits the child demonstrates normal growth and development. Prior to discharge the parents state that they understand the medication regimen. The parents fill the prescription for hormone replacement therapy prior to discharge. The parents ask appropriate questions about the planned treatment goals.

During follow-up visits the child demonstrates normal growth and development. 21-OH enzyme deficiency results in blocking the production of adrenal mineralocorticoids and glucocorticoids. Nursing management of the infant or child with CAH focuses on preventing and monitoring for acute adrenal crisis, helping the family to understand the disease, providing education to the child and family about the importance of maintaining hormone supplementation, and providing emotional support to the family. Improvement of symptoms, such as normal growth and development, is the best indicator that the goal of hormone replacement therapy is being carried out as ordered.

A preschooler has celiac disease. The parent is preparing a gluten-free diet. The nurse knows that the parent understands the diet when the parent prepares which breakfast foods? Rye toast and peanut butter Wheat toast and grape jelly Cheerios (oat cereal) and skim milk Eggs and orange juice

Eggs and orange juice

The mother of a newborn with a cleft lip reports she is having a hard time looking her baby. What is the best action by the nurse? Encourage the child's mother to hold her infant against her shoulder to provide closeness while not looking at the defect. Explain that surgery will make this better in the future. Tell the mother that while this is difficult it will get easier. Encourage the mother to provide care for her infant.

Encourage the mother to provide care for her infant. Providing care to the infant is the best means for the mother to begin bonding with her baby. Activities such as feeding, diapering and bathing will be helpful.

The nurse is caring for a client suspected of having stones that have collected in the common bile duct. What test should the nurse prepare the client for that will locate these stones? Cholecystectomy Endoscopic retrograde cholangiopancreatography (ERCP) Abdominal x-ray Colonoscopy

Endoscopic retrograde cholangiopancreatography (ERCP) Explanation: ERCP locates stones that have collected in the common bile duct. A colonoscopy will not locate gallstones but only allows visualization of the large intestine. Abdominal x-ray is not a reliable locator of gallstones. A cholecystectomy is the surgical removal of the gallbladder.

A newborn is discovered to have congenital adrenogenital hyperplasia. What will the nurse most likely observe when assessing this client? Abnormal facial features Divergent vision Enlarged clitoris Small for gestational age

Enlarged clitoris Congenital adrenal hyperplasia is a syndrome that is inherited as an autosomal recessive trait, which causes the adrenal glands to not be able to synthesize cortisol. Because the adrenal gland is unable to produce cortisol, the level of adrenocorticotropic hormone (ACTH) secreted by the pituitary in an attempt to stimulate the gland to increase function is increased.

A physician orders spironolactone (Aldactone), 50 mg by mouth four times daily, for a client with fluid retention caused by cirrhosis. Which finding indicates that the drug is producing a therapeutic effect? Loss of 2.2 lb (1 kg) in 24 hours Serum potassium level of 3.5 mEq/L Serum sodium level of 135 mEq/L Blood pH of 7.25

Explanation: Daily weight measurement is the most accurate indicator of fluid status; a loss of 2.2 lb (1 kg) indicates loss of 1 L of fluid. Because spironolactone is a diuretic, weight loss is the best indicator of its effectiveness. This client's serum potassium and sodium levels are normal. A blood pH of 7.25 indicates acidosis, an adverse reaction to spironolactone.

A patient sustained an open fracture of the femur 24 hours ago. While assessing the patient, the nurse observes the patient is having difficulty breathing, and oxygen saturation decreases to 88% from a previous 99%. What does the nurse understand is likely occurring with this patient? Pneumonia Spontaneous pneumothorax Cardiac tamponade Fat emboli

Fat emboli Explanation: After fracture of long bones or pelvic bones, or crush injuries, fat emboli frequently form. Fat embolism syndrome (FES) occurs when fat emboli cause morbid clinical manifestations. The classic triad of clinical manifestations of FES include hypoxemia, neurologic compromise, and a petechial rash (NAON, 2007), although not all signs and symptoms manifest at the same time (Tzioupis & Giannoudis, 2011). The typical first manifestations are pulmonary and include hypoxia and tachypnea.

The client with a fractured left humerus reports dyspnea and chest pain. Pulse oximetry is 88%. Temperature is 100.2 degrees Fahrenheit (38.5 degrees Centigrade); heart rate is 110 beats per minute; respiratory rate is 32 breaths per minute. The nurse suspects the client is experiencing: Compartment syndrome Fat embolism syndrome Complex regional pain syndrome Delayed union

Fat embolism syndrome Explanation: The clinical manifestations described in the scenario are characteristic of fat embolism syndrome.

The nurse is caring for a client with hepatocellular jaundice. Which finding(s) will the nurse expect to assess in this client? Select all that apply. Fatigue Nausea Malaise Weight gain Constipation

Fatigue Nausea Malaise Hepatocellular jaundice is caused by the inability of damaged liver cells to clear normal amounts of bilirubin from the blood. The cellular damage may be caused by hepatitis viruses, other viruses that affect the liver, chemical toxins, or alcohol. Clients with hepatocellular jaundice may be mildly or severely ill, and experience fatigue, nausea, and malaise. Weight loss may occur because of a lack of appetite. Constipation is not associated with hepatocellular jaundice.

A parent brings the 2-week-old infant to the office because the infant has been experiencing gastroesophageal reflux over the past week. Which intervention(s) should the nurse recommend to the parent at this point? Select all that apply. If breastfeeding, switch to feeding the infant formula. Consult the heath care provider regarding having botulinum toxin injected into the lower esophageal sphincter. Consult a pediatric surgeon regarding having a myotomy procedure performed. Feed the infant while holding the infant in an upright position. Keep the infant upright by holding them and/or elevating the head of the crib after feeding. Feed the infant a formula thickened with rice cereal.

Feed the infant a formula thickened with rice cereal. Feed the infant while holding the infant in an upright position. Keep the infant upright by holding them and/or elevating the head of the crib after feeding. The traditional treatment of gastroesophageal reflux in the infant is to feed a formula thickened with rice cereal (1 tbsp of cereal per 1 oz of formula or breast milk) while holding the infant in an upright position and then keeping the infant upright by holding them and/or elevating the head of the crib 30 degrees for 30 to 45 minutes after feeding so gravity can help prevent reflux.

Which type of deficiency results in macrocytic anemia? Vitamin C Vitamin K Vitamin A Folic acid

Folic acid Explanation: Folic acid deficiency results in macrocytic anemia. Vitamin C deficiency results in hemorrhagic lesions of scurvy. Vitamin A deficiency results in night blindness and eye and skin changes. Vitamin K deficiency results in hypoprothrombinemia, which is characterized by spontaneous bleeding and ecchymosis.

A client is brought in by ambulance to the emergency department after being involved in a motorcycle accident. The client has an open fracture on his tibia. The wound is highly contaminated and there is extensive soft- tissue damage. How would this client's fracture likely be graded? Grade IV Grade II Grade I Grade III

Grade III Explanation: Open fractures are graded according to the following criteria. Grade I is a clean wound less than 1 cm long. Grade II is a larger wound without extensive soft-tissue damage. Grade III is highly contaminated, has extensive soft-tissue damage, and is the most severe. There is no grade IV fracture.

During an assessment of an adolescent child, the nurse notes that the child has a protuberant tongue, fatigued appearance, poor muscle tone, and exophthalmos. What medical diagnosis would the nurse expect the child to have? Cushing disease Graves disease syndrome of inappropriate antidiuretic hormone secretion (SIADH)

Graves disease Symptoms of Graves disease include an increased rate of growth; weight loss despite an excellent appetite; hyperactivity; warm, moist skin; tachycardia; fine tremors; an enlarged thyroid gland or goiter; and ophthalmic changes including exophthalmos. These are not symptoms of Cushing disease, diabetes, or SIADH.

A 12-year-old child has hyperthyroidism. The nurse understands that the most common cause of hyperthyroidism in children is: Plummer disease. Graves disease. Cushing syndrome. Addison disease.

Graves disease. Hyperthyroidism occurs less often in children than hypothyroidism. Graves disease is the most common cause of hyperthyroidism in children. Hyperthyroidism occurs more often in females, and the peak incidence occurs during adolescence. Addison disease refers to chronic adrenocortical insufficiency. Cushing syndrome results from excessive levels of circulating cortisol. Plummer disease is a less common cause of hyperthyroidism.

A patient in pelvic traction needs circulatory status assessed. How should the nurse assess for a positive Homans' sign? Have the patient plantar flex both feet while the nurse performs the blanch test on all of the patient's toes. Have the patient squeeze the nurse's hands with his or her hands to evaluate any difference in strength. Have the patient extend each leg and dorsiflex each foot to determine if pain or tenderness is present in the lower leg. Have the patient extend both hands while the nurse compares the volume of both radial pulses.

Have the patient extend each leg and dorsiflex each foot to determine if pain or tenderness is present in the lower leg. Explanation: The nurse should assess for pain on passive flexion of each foot, which could indicate deep vein thrombosis.

A client has just undergone a leg amputation. What will the nurse closely monitor the client for during the immediate postoperative period? Unexplainable burning pain (causalgia) Hematoma Neuroma Chronic osteomyelitis

Hematoma Explanation: Hematoma, hemorrhage, and infection are potential complications in the immediate postoperative period. Sleeplessness, nausea, and vomiting may occur but are adverse reactions, not complications. Chronic osteomyelitis and causalgia are potential complications that are likely to arise in the late postoperative period. A neuroma occurs when the cut ends of the nerves become entangled in the healing scar. This would occur later in the postoperative course.

The nurse identifies which type of jaundice in an adult experiencing a transfusion reaction? Hepatocellular Hemolytic Nonobstructive Obstructive

Hemolytic Hemolytic jaundice occurs because, although the liver is functioning normally, it cannot excrete the bilirubin as quickly as it is formed. This type of jaundice is encountered in clients with hemolytic transfusion reactions and other hemolytic disorders. Obstructive and hepatocellular jaundice are the result of liver disease. Nonobstructive jaundice occurs with hepatitis.

Which type of jaundice seen in adults is the result of increased destruction of red blood cells? Hemolytic Nonobstructive Hepatocellular Obstructive

Hemolytic Hemolytic jaundice results because, although the liver is functioning normally, it cannot excrete the bilirubin as quickly as it is formed. Obstructive and hepatocellular jaundice are results of liver disease. Nonobstructive jaundice occurs with hepatitis.

A client has an elevated serum ammonia concentration and is exhibiting changes in mental status. The nurse should suspect which condition? Cirrhosis Portal hypertension Asterixis Hepatic encephalopathy

Hepatic encephalopathy Hepatic encephalopathy is a central nervous system dysfunction resulting from liver disease. It is frequently associated with an elevated ammonia concentration that produces changes in mental status, altered level of consciousness, and coma. Portal hypertension is an elevated pressure in the portal circulation resulting from obstruction of venous flow into and through the liver. Asterixis is an involuntary flapping movement of the hands associated with metabolic liver dysfunction.

A nurse is preparing a presentation for a local community group about hepatitis. Which of the following would the nurse include? Infection with hepatitis G is similar to hepatitis A. Hepatitis B is transmitted primarily by the oral-fecal route. Hepatitis C increases a person's risk for liver cancer. Hepatitis A is frequently spread by sexual contact.

Hepatitis C increases a person's risk for liver cancer. Infection with hepatitis C increases the risk of a person developing hepatic (liver) cancer. Hepatitis A is transmitted primarily by the oral-fecal route; hepatitis B is frequently spread by sexual contact and infected blood. Hepatitis E is similar to hepatitis A whereas hepatitis G is similar to hepatitis C.

A client has developed drug-induced hepatitis from a drug reaction to antidepressants. What treatment does the nurse anticipate the client will receive to treat the reaction? Liver transplantation High-dose corticosteroids Paracentesis Azathioprine

High-dose corticosteroids Explanation: Drug-induced hepatitis occurs when a drug reaction damages the liver. This form of hepatitis can be severe and fatal. High-dose corticosteroids usually administered first to treat the reaction. Liver transplantation may be necessary. Paracentesis would be used to withdraw fluid for the treatment of ascites. Azathioprine (Imuran) may be used for autoimmune hepatitis.

Which factor inhibits fracture healing? History of diabetes Increased vitamin D and calcium in the diet Age of 35 years Immobilization of the fracture

History of diabetes Explanation: Factors that inhibit fracture healing include diabetes, smoking, local malignancy, bone loss, extensive local trauma, age greater than 40, and infection. Factors that enhance fracture healing include proper nutrition, vitamin D and calcium, exercise, maximum bone fragment contact, proper alignment, and immobilization of the fracture.

A nurse caring for a child with Graves disease is administering propylthiouracil (PTU). The child has been on this drug for a few weeks and now has sudden symptoms of a sore throat. What is the priority intervention for the nurse? Offer throat lozenges to soothe the throat. Ask the child if there is a reason he or she does not want to go back to school. Continue medication to relieve the signs of Graves disease. Hold the dose and call the health care provider.

Hold the dose and call the health care provider. The severe sore throat could be a sign of leukopenia, which is a side effect of PTU. The medication should be held and the health care provider called. The medication dose may need to be adjusted. Lozenges will not help this side effect. It is not appropriate to imply that a child may be making up symptoms to avoid school.

Clients diagnosed with esophageal varices are at risk for hemorrhagic shock. Which is a sign of potential hypovolemia? Polyuria Warm moist skin Hypotension Bradycardia

Hypotension Signs of potential hypovolemia include cool, clammy skin; tachycardia; decreased blood pressure; and decreased urine output.

The nurse is monitoring a patient who sustained a fracture of the left hip. The nurse should be aware that which kind of shock can be a complication of this type of injury? Cardiogenic Septic Hypovolemic Neurogenic

Hypovolemic Explanation: In a client with a pelvic fracture, the nurse should be aware of the potential for hypovolemic shock resulting from hemorrhage. Cardiogenic shock, in which the heart cannot pump enough blood to meet the body's needs, often arises from severe myocardial infarction. Neurogenic shock is often a consequence of spinal cord injury and resulting loss of sympathetic nervous system function. Septic shock results from body-wide infection.

The nurse is caring for a child with a diagnosis of pyloric stenosis during the preoperative phase of the child's treatment. What is the highest priority at this time? Improving hydration Maintaining skin integrity Promoting comfort Preparing family for home care

Improving hydration Preoperatively, the highest priority for the child with pyloric stenosis is to improve nutrition and hydration. Maintaining mouth and skin integrity, and relieving family anxiety are important, but these are not the priority. The child will not likely have intense pain. Preparing the family for home care would be a postoperative goal.

A nurse is assigned to support a patient while a cast is being applied to treat a greenstick fracture. The nurse documents that this fracture is classified as what type of fracture? Incomplete Compression Stress Closed

Incomplete Explanation: A greenstick fracture involves a break through only part of the cross-section of the bone.

Which type of fracture involves a break through only part of the cross-section of the bone? Incomplete Comminuted Oblique Open

Incomplete Explanation: An incomplete fracture involves a break through only part of the cross-section of the bone. A comminuted fracture is one that produces several bone fragments. An open fracture is one in which the skin or mucous membrane wound extends to the fractured bone. An oblique fracture runs across the bone at a diagonal angle of 45 to 60 degrees.

Which nursing diagnosis takes highest priority for a client with a compound fracture? Infection related to effects of trauma Impaired physical mobility related to trauma Imbalanced nutrition: Less than body requirements related to immobility Activity intolerance related to weight-bearing limitations

Infection related to effects of trauma Explanation: A compound fracture involves an opening in the skin at the fracture site. Because the skin is the body's first line of defense against infection, any skin opening places the client at risk for infection. Imbalanced nutrition: Less than body requirements is rarely associated with fractures. Although Impaired physical mobility and Activity intolerance may be associated with any fracture, these nursing diagnoses don't take precedence because they aren't as life-threatening as infection.

The nurse is caring for a 10-year-old child with growth hormone (GH) deficiency. Which therapy would you anticipate will be prescribed for the child? Short-term aldosterone provocation Injections of GH Long-term blocking of beta cells Oral administration of somatotropin

Injections of GH Growth hormone (GH) deficiency occurs when the anterior pituitary is unable to produce enough hormone for usual growth. Somatotropin is the name of the growth hormone administered. Administering subcutaneous GH to the child helps correct this deficiency. The GH dosage is 0.2 to 0.3 mg/kg given daily. It is not administered orally. Aldosterone causes sodium to be retained and a provocation would be the administration of diuretics to reduce the sodium. Beta cells are found in the heart muscles, smooth muscles, airways, and arteries. They are also found in the pancreas to secrete insulin.

Which intervention should the nurse implement with the client who has undergone a hip replacement? Adduct the legs by placing a pillow between the legs. Instruct the client to avoid internal rotation of the leg. Have the client bend forward to rise from the chair. Place the client in high Fowler's position for meals.

Instruct the client to avoid internal rotation of the leg. Explanation: The client should avoid all activities that can result in dislocation of the hip. The affected leg should not cross midline or be turned inward. A pillow is used to keep the legs in abduction. The hip should not bend more than 90 degrees when seated. The head of bed should be kept at 60 degrees or less.

The nurse is administering medications to a client that has elevated ammonia due to cirrhosis of the liver. What medication will the nurse give to detoxify ammonium and to act as an osmotic agent? Lactulose Kanamycin Cholestyramine Spironolactone

Lactulose Lactulose is administered to detoxify ammonium and to act as an osmotic agent, drawing water into the bowel, which causes diarrhea in some clients. Potassium-sparing diuretics such as spironolactone are used to treat ascites. Cholestyramine is a bile acid sequestrant and reduces pruritus. Kanamycin decreases intestinal bacteria and decreases ammonia but does not act as an osmotic agent.

A 9-month-old girl is brought to the emergency room with what appears to be bouts of intense abdominal pain 15 minutes apart in which she draws up her legs and cries, often accompanied by vomiting. In between the bouts, the child recovers and appears to be without symptoms. Blood is found in the stool. What condition should the nurse suspect in this case? Short-bowel/short-gut syndrome Necrotizing enterocolitis Volvulus with malrotation Intussusception

Intussusception Intussusception, the invagination of one portion of the intestine into another, usually occurs in the second half of the first year of life. Children with this disorder suddenly draw up their legs and cry as if they are in severe pain; they may vomit. After the peristaltic wave that caused the discomfort passes, they are symptom-free and play happily. In approximately 15 minutes, however, the same phenomenon of intense abdominal pain strikes again. After approximately 12 hours, blood appears in the stool and possibly in vomitus, described as a "currant jelly" appearance. Volvulus with malrotation and necrotizing enterocolitis typically occur in the first 6 months of life and do not match the symptoms described above. Short-bowel/short-gut syndrome typically occurs when a large portion of the intestine has been removed due to a previous disease or trauma.

While conducting a physical examination of a client, which of the following skin findings would alert the nurse to the liklihood of liver problems? Select all that apply. Aphthous stomatitis Ecchymoses Cyanosis of the lips Jaundice Petechiae

Jaundice Petechiae Ecchymoses Explanation: The skin, mucosa, and sclerae are inspected for jaundice. The nurse observes the skin for petechiae or ecchymotic areas (bruises), spider angiomas, and palmar erythema. Cyanosis of the lips is indicative of a problem with respiratory or cardiovascular dysfunction. Aphthous stomatitis is a term for mouth ulcers and is a gastrointestinal abnormal finding.

A child is diagnosed with diabetes insipidus (DI) and will start on the medication desmopressin. Which instruction(s) would the nurse include in teaching the parent about administering this drug? Select all that apply. Clear the nostrils before administering the drug. Keep the drug refrigerated. If the child sneezes, repeat the dose administration. The drug must be administered every 24 hours. Monitor the urine specific gravity while the child is taking the drug.

Keep the drug refrigerated. Clear the nostrils before administering the drug. If the child sneezes, repeat the dose administration. Monitor the urine specific gravity while the child is taking the drug.

A client is being prepared to undergo laboratory and diagnostic testing to confirm the diagnosis of cirrhosis. Which test would the nurse expect to be used to provide definitive confirmation of the disorder? Magnetic resonance imaging Liver biopsy Radioisotope liver scan Coagulation studies

Liver biopsy A liver biopsy which reveals hepatic fibrosis is the most conclusive diagnostic procedure. Coagulation studies provide information about liver function but do not definitively confirm the diagnosis of cirrhosis. Magnetic resonance imaging and radioisotope liver scan help to support the diagnosis but do not confirm it. These tests provide information about the liver's enlarged size, nodular configuration, and distorted blood flow.

A previously healthy adult's sudden and precipitous decline in health has been attributed to fulminant hepatic failure, and the client has been admitted to the intensive care unit. The nurse should be aware that the treatment of choice for this client is what? Lobectomy IV administration of immune globulins Liver transplantation Transfusion of packed red blood cells and fresh-frozen plasma (FFP)

Liver transplantation Explanation: Liver transplantation carries the highest potential for the resolution of fulminant hepatic failure. This is preferred over other interventions, such as pharmacologic treatments, transfusions, and surgery.

A client who was in a motor vehicle crash is diagnosed with a stable T7 spinal fracture with no neurologic deficits. Which nursing intervention should the nurse implement? Withhold opioid pain medication to prevent ileus. Maintain bed rest with the head of the bed at 20 degrees. Maintain NPO (nothing by mouth) status for surgical repair. Sit the client upright in a padded chair for meals.

Maintain bed rest with the head of the bed at 20 degrees. Explanation: The client should maintain limited bed rest with the head of the bed lower than 30 degrees. If the client's pain is not controlled with a lower form of pain medication, then an opioid may be used to treat the pain. The nurse should monitor for an ileus. Stable spinal fractures are treated conservatively and not with surgical repair. The client should avoid sitting until the pain eases.

A client with a fractured femur is in balanced suspension traction. The client needs to be repositioned toward the head of the bed. During repositioning, what should the nurse do? Release the weights and replace them immediately after positioning. Place slight additional tension on the traction cords. Maintain consistent traction tension while repositioning. Reposition the bed instead of repositioning the client.

Maintain consistent traction tension while repositioning. Explanation: Traction is used to reduce the fracture and must be maintained at all times, including during repositioning. It would be inappropriate to add tension or release the weights. Moving the bed instead of the client is not feasible.

A client with cirrhosis has a massive hemorrhage from esophageal varices. Balloon tamponade is used temporarily to control hemorrhage and stabilize the client. In planning care, the nurse gives the highest priority to which goal? Maintaining fluid volume Relieving the client's anxiety Maintaining the airway Controlling bleeding

Maintaining the airway Explanation: Esophageal varices are almost always caused by portal hypertension, which results from obstruction of the portal circulation within the damaged liver. Maintaining the airway is the highest priority because oxygenation is essential for life. The airway can be compromised by possible displacement of the tube and the inflated balloon into the oropharynx, which can cause life-threatening obstruction of the airway and asphyxiation.

When performing a physical examination on a client with cirrhosis, a nurse notices that the client's abdomen is enlarged. Which of the following interventions should the nurse consider? Measure abdominal girth according to a set routine. Ask the client about food intake. Provide the client with nonprescription laxatives. Report the condition to the physician immediately.

Measure abdominal girth according to a set routine. Explanation: If the abdomen appears enlarged, the nurse measures it according to a set routine. The nurse reports any change in mental status or signs of gastrointestinal bleeding immediately. It is not essential for the client to take laxatives unless prescribed. The client's food intake does not affect the size of the abdomen in case of cirrhosis.

Which term refers to the failure of fragments of a fractured bone to heal together? Nonunion Subluxation Dislocation Malunion

Nonunion When nonunion occurs, the client reports persistent discomfort and movement at the fracture site. Dislocation refers to the separation of joint surfaces. Subluxation refers to partial separation or dislocation of joint surfaces. Malunion refers to growth of the fragments of a fractured bone in a faulty position, forming an imperfect union.

The nurse assesses a client after total right hip arthroplasty and observes a shortening of the extremity. The client reports severe pain in the right side of the groin. What is the priority action of the nurse? Apply Buck's traction. Externally rotate the extremity. Notify the health care provider. Bend the knee and rotate the knee internally.

Notify the health care provider. Explanation: If any clinical manifestations of dislocation of the prosthesis occur, including acute groin pain in the affected hip or shortening of the affected extremity, the nurse (or the client, if at home) must immediately notify the surgeon, because the hip must be reduced and stabilized promptly so that the leg does not sustain circulatory and nerve damage. After closed reduction, the hip may be stabilized with Buck's traction or a brace to prevent recurrent dislocation. As the muscles and joint capsule heal, the chance of dislocation diminishes. Stresses to the new hip joint should be avoided for the first 8 to 12 weeks, when the risk of dislocation is greatest.

A client is actively bleeding from esophageal varices. Which medication would the nurse most expect to be administered to this client? Octreotide Propranolol Lactulose Spironolactone

Octreotide In an actively bleeding client, medications are administered initially because they can be obtained and administered quicker than other therapies. Octreotide (Sandostatin) causes selective splanchnic vasoconstriction by inhibiting glucagon release and is used mainly in the management of active hemorrhage.

A client has undergone a liver biopsy. After the procedure, the nurse should place the client in which position? On the right side Trendelenburg On the left side High Fowler

On the right side Explanation: Immediately after the biopsy, assist the client to turn on to the right side; place a pillow under the costal margin, and caution the client to remain in this position. In this position, the liver capsule at the site of penetration is compressed against the chest wall, and the escape of blood or bile through the perforation made for the biopsy is impeded. Positioning the client on the left side is not indicated. Positioning the client in the Trendelenburg position may be indicated if the client is in shock, but it is not the position designed for the client after liver biopsy. The high Fowler position is not indicated for the client after liver biopsy.

A client is brought to the emergency department after injuring the right arm in a bicycle accident. The orthopedic surgeon tells the nurse that the client has a greenstick fracture of the arm. What does this mean? Bone fragments are separated at the fracture line. The fracture line extends through the entire bone substance. The fracture results from an underlying bone disorder. One side of the bone is broken and the other side is bent.

One side of the bone is broken and the other side is bent. Explanation: In a greenstick fracture, one side of the bone is broken and the other side is bent. A greenstick fracture also may refer to an incomplete fracture in which the fracture line extends only partially through the bone substance and doesn't disrupt bone continuity completely. (Other terms for greenstick fracture are willow fracture and hickory-stick fracture.) The fracture line extends through the entire bone substance in a complete fracture. A fracture that results from an underlying bone disorder, such as osteoporosis or a tumor, is a pathologic fracture, which typically occurs with minimal trauma. Bone fragments are separated at the fracture line in a displaced fracture.

The nurse is caring for a teenager diagnosed with acute pancreatitis. Which order would the nurse question? PO pain management NPO nasogastric tube placed to suction serum amylase levels

PO pain management Maintain NPO status and nasogastric tube suction and patency. Administer intravenous fluids to keep the child hydrated and correct any alterations in fluid and electrolyte balance. Pain management is crucial in children with pancreatitis; due to NPO status, medications are typically prescribed intravenously. Serial monitoring of serum amylase levels will determine when oral feeding may be restarted.

A client with metastatic bone cancer sustained a left hip fracture without injury. What type of fracture does the nurse understand occurs without trauma or fall? Compound fracture Impacted fracture Transverse fracture Pathologic fracture

Pathologic fracture Explanation: A pathologic fracture is a fracture that occurs through an area of diseased bone and can occur without trauma or a fall. An impacted fracture is a fracture in which a bone fragment is driven into another bone fragment. A transverse fracture is a fracture straight across the bone. A compound fracture is a fracture in which damage also involves the skin or mucous membranes.

In caring for an infant diagnosed with pyloric stenosis the nurse would anticipate which intervention? Prepare the infant for surgery. Assist in insertion of a nasogastric (NG) tube. Change the infant's diet to one that is lactose-free. Assist in doing a barium enema procedure on the infant.

Prepare the infant for surgery. In pyloric stenosis, the thickened muscle of the pylorus causes gastric outlet obstruction. The treatment is a surgical correction called a pyloromyotomy. The condition is not related to lactose in the diet, so changing to lactose-free formula would not correct the condition. A barium enema would be used to diagnose intussusception. A nasogastric (NG) tube is inserted for gastric decompression in an infant with intussusception.

The nurse is caring for a patient who had a total hip replacement. What lethal postoperative complication should the nurse closely monitor for? Urinary tract infection Atelectasis Pulmonary embolism Hypovolemia

Pulmonary embolism Explanation: Patients having orthopedic surgery are particularly at risk for venous thromboembolism, including deep vein thrombosis and pulmonary embolism.

The nurse is caring for a client with cirrhosis. Which assessment findings indicate that the client has deficient vitamin K absorption caused by this hepatic disease? Purpura and petechiae Dyspnea and fatigue Gynecomastia and testicular atrophy Ascites and orthopnea

Purpura and petechiae A hepatic disorder, such as cirrhosis, may disrupt the liver's normal use of vitamin K to produce prothrombin (a clotting factor). Consequently, the nurse should monitor the client for signs of bleeding, including purpura and petechiae. Dyspnea and fatigue suggest anemia. Ascites and orthopnea are unrelated to vitamin K absorption. Gynecomastia and testicular atrophy result from decreased estrogen metabolism by the diseased liver.

A client with cirrhosis has portal hypertension, which is causing esophageal varices. What is the goal of the interventions that the nurse will provide? Treat the esophageal varices. Promote optimal neurologic function. Cure the cirrhosis. Reduce fluid accumulation and venous pressure.

Reduce fluid accumulation and venous pressure. Explanation: Methods of treating portal hypertension aim to reduce fluid accumulation and venous pressure. There is no cure for cirrhosis; treating the esophageal varices is only a small portion of the overall objective. Promoting optimal neurologic function will not reduce portal hypertension.

After undergoing a liver biopsy, a client should be placed in which position? Semi-Fowler's position Prone position Right lateral decubitus position Supine position

Right lateral decubitus position Explanation: After a liver biopsy, the client is placed on the right side (right lateral decubitus position) to exert pressure on the liver and prevent bleeding. Semi-Fowler's position and the supine and prone positions wouldn't achieve this goal.

In what location would the nurse palpate for the liver? Left upper quadrant Right upper quadrant Right lower quadrant Left lower quadrant

Right upper quadrant The liver may be palpable in the right upper quadrant. A palpable liver presents as a firm, sharp ridge with a smooth surface.

A client arrives in the emergency department with a suspected bone fracture of the right arm. How does the nurse expect the client to describe the pain? Similar to "muscle cramps" Sharp and piercing Sore and aching A dull, deep, boring ache

Sharp and piercing

hich is an inaccurate principle of traction? The client must be in good alignment in the center of the bed. Skeletal traction is interrupted to turn and reposition the client. The weights are not removed unless intermittent treatment is prescribed. The weights must hang freely.

Skeletal traction is interrupted to turn and reposition the client. Explanation: Skeletal traction is never interrupted. The weights are not removed unless intermittent treatment is prescribed. The weights must hang freely, with the client in good alignment in the center of the bed.

The nurse is caring for a 6-month-old infant with diarrhea and dehydration. The parent is concerned because the infant has some patches on the tongue. Which feature indicates a geographic tongue? There are also white patches on the erupted teeth. There are also plaques on the buccal mucosa. Some patches are light in color and other patches are dark in color. The patches are thick, white plaques on the tongue.

Some patches are light in color and other patches are dark in color. A geographic tongue is a benign, noncontagious condition characterized by a reduction in the filiform papillae. Oral candidiasis (thrush) is characterized by thick, white plaques that form on the tongue. With thrush, plaques also appear on the buccal mucosa and often occur concomitantly in the diaper area. There would not be any patches on the few teeth the infant may have by that age.

The nurse is concerned about potassium loss when a diuretic is prescribed for a patient with ascites and edema. What diuretic may be ordered that spares potassium and prevents hypokalemia? Acetazolamide (Diamox) Furosemide (Lasix) Bumetanide (Bumex) Spironolactone (Aldactone)

Spironolactone (Aldactone) Potassium-sparing diuretic agents such as spironolactone or triamterene (Dyrenium) may be indicated to decrease ascites, if present; these diuretics are preferred because they minimize the fluid and electrolyte changes commonly seen with other agents.

An adolescent has hepatitis B. What would be the most important nursing action? Strict enforcement of standard precautions Close observation to detect cerebral hallucinations Conscientious collection of stool for ova and parasites Strict calculation of caloric and vitamin B intake

Strict enforcement of standard precautions Hepatitis B is spread through IV drug use, sex, contaminated blood and perinatally. The treatment is rest, hydration, and nutrition. Hospitalization is required if there is vomiting, dehydration, elevated bleeding times and mental status changes. The adolescent should be taught about good hygiene, safe sex practices, careful handwashing and blood/bodily fluid contact precautions. Using standard precautions of gloves and good handwashing will help prevent spread of the disease.

A client with hepatitis who has not responded to medical treatment is scheduled for a liver transplant. Which of the following most likely would be ordered? Chenodiol Ursodiol Tacrolimus Interferon alfa-2b, recombinant

Tacrolimus In preparation for a liver transplant, a client receives immunosuppressants to reduce the risk for organ rejection. Tacrolimus or cyclosporine are two immunosuppressants that may be used. Chenodiol and ursodiol are agents used to dissolve gall stones. Recombinant interferon alfa-2b is used to treat chronic hepatitis B, C, and D to force the virus into remission.

The nurse is caring for a 6-month-old infant who was admitted to the emergency department 24 hours ago with signs of severe diarrhea. The infant's rectal temperature is 104°F (40°C), with weak and rapid pulse and respirations. The skin is pale and cool. The child is on IV rehydration therapy, but the diarrhea is persisting. The infant has not voided since being admitted. Which is the priority nursing intervention? Take a stool culture Administer antibiotic therapy Administer IV potassium Feed the child a cracker

Take a stool culture Treatment of severe diarrhea focuses on regulating electrolyte and fluid balance by initiating a temporary rest for the gastrointestinal tract, oral or IV rehydration therapy, and discovering the organism responsible for the diarrhea. All children with severe diarrhea or diarrhea that persists longer than 24 hours should have a stool culture taken to determine if bacteria are causing the diarrhea; if so, a definite antibiotic therapy can be prescribed. Because a side effect of many antibiotics is diarrhea, antibiotics should not routinely be used to treat diarrhea without an identifiable bacterial cause. Before the initial IV fluid is changed to a potassium solution, be certain the infant or child has voided—proof that the kidneys are functioning; in this case, the child is not voiding yet. The child should not be fed a cracker, as the gastrointestinal tract should be rested until the diarrhea stops.

The nurse is assessing a 10-day-old infant for dehydration. Which finding indicates severe dehydration? Tenting of skin Soft and flat fontanels (fontanelles) Pale and slightly dry mucosa Blood pressure of 80/42 mm Hg

Tenting of skin

An adolescent is diagnosed with hepatitis A. Which problem should be considered when planning care? The adolescent's urine will be dark and infectious. The adolescent will become fatigued easily. Hypothermia is common. The adolescent will be very irritable and perhaps require sedation.

The adolescent will become fatigued easily. Hepatitis A is transmitted via the oral-fecal route; it is water borne and often occurs in areas of poor sanitation. The adolescent with hepatitis A will exhibit flu-like symptoms, a headache, anorexia and fatigue. The urine is not infectious and fever may be present as opposed to hypothermia. Irritability is not one of the symptoms of hepatitis A. The client is usually lethargic or listless.

A 9-year-old child with Graves disease is seen at the pediatrician's office reporting sore throat and fever. The nurse notes in the history that the child is taking propylthiouracil. Which of the following would concern the nurse? The child needs to be started on an antibiotic drug. The child may have developed leukopenia. The child must be participating in sports. The child may not be taking the medication.

The child may have developed leukopenia. Graves disease is defined as an overproduction of thyroid hormones. Propylthiouracil is used to suppress thyroid function. A complication of Graves disease is leukopenia.

The nurse is assessing an 8-year-old boy who is performing academically at a second-grade level. The mother reports that the boy states feeling weak and tired and has had a weight increase of 6 pounds (13.2 kg) in 3 months. Which additional data would fit with a possible diagnosis of hypothyroidism? The mother reports that the boy is always thirsty. Oral cavity assessment shows two of the 6-year molars. The child states that the exam room is cold. The child has a faint rash on the trunk of the body.

The child states that the exam room is cold. Cold intolerance, manifested by the fact that the child was uncomfortably cold in the exam room, is a sign of hypothyroidism. Delayed dentition, with only two of the four 6-year molars having erupted, is typical of growth hormone deficiency. Complaints of thirst may signal diabetes or diabetes insipidus. A rash can be varied disease processes but is not characteristic in hypothyroidism.

A nurse is caring for a client with cirrhosis. The nurse assesses the client at noon and discovers that the client is difficult to arouse and has an elevated serum ammonia level. The nurse should suspect which situation? The client didn't take his morning dose of lactulose (Cephulac). The client is avoiding the nurse. The client is relaxed and not in pain. The client's hepatic function is decreasing.

The decreased level of consciousness caused by an increased serum ammonia level indicates hepatic disfunction. If the client didn't take his morning dose of lactulose, he wouldn't have elevated ammonia levels and decreased level of consciousness this soon. These assessment findings don't indicate that the client is relaxed or avoiding the nurse.

Ammonia, the major etiologic factor in the development of encephalopathy, inhibits neurotransmission. Increased levels of ammonia are damaging to the body. The largest source of ammonia is from: The digestion of dietary and blood proteins. Excessive diuresis and dehydration. Severe infections and high fevers. Excess potassium loss subsequent to prolonged use of diuretics.

The digestion of dietary and blood proteins. Explanation: Circumstances that increase serum ammonia levels tend to aggravate or precipitate hepatic encephalopathy. The largest source of ammonia is the enzymatic and bacterial digestion of dietary and blood proteins in the GI tract. Ammonia from these sources increases as a result of GI bleeding (i.e., bleeding esophageal varices, chronic GI bleeding), a high-protein diet, bacterial infection, or uremia.

An elite high school football player has been diagnosed and treated for a shoulder dislocation. What should the nurse emphasize during health education to facilitate the player's rejoining the team? The fact that he has a permanently increased risk of future shoulder dislocations The importance of monitoring for intracapsular bleeding once he resumes playing The need to take analgesia regardless of the short-term absence of pain The importance of adhering to the prescribed treatment and rehabilitation regimen

The importance of adhering to the prescribed treatment and rehabilitation regimen Explanation: Clients who have experienced sports-related injuries are often highly motivated to return to their previous level of activity. Adherence to restriction of activities and gradual resumption of activities need to be reinforced. Appropriate analgesia use must be encouraged, but analgesia does not necessarily have to be taken in the absence of pain. If healing is complete, the client does not likely have a greatly increased risk of reinjury. Dislocations rarely cause bleeding after the healing process.

The nurse is preparing a client to have his cast cut off after having it for 6 weeks to treat a fractured tibia. What should the nurse inform the client prior to the cast being removed? The leg will look as it did prior to the cast being applied. The skin may be covered with a yellowish crust that will shed in a few days. The leg strength is enforced by the wearing of the cast. The leg will look moist and will have small bumps that will go away in a few days.

The skin may be covered with a yellowish crust that will shed in a few days. Explanation: Once the cast is off, the skin appears mottled and may be covered with a yellowish crust composed of accumulated body oil and dead skin. The client usually sheds this residue in a few days. The leg will not look as it did prior to the cast but will regain the same shape and status as the other leg. There should be no bumps underneath the cast. The leg may be weak and stiff for some time after the cast is removed, not stronger.

What occurs in the gastrointestinal system of the child with Hirschsprung disease? There is an invagination or telescoping of one portion of the bowel into a distal portion. There is a partial or complete mechanical obstruction in the intestine. There is a severe narrowing of the lumen of the pylorus. There is a relaxed sphincter in the lower portion of the esophagus.

There is a partial or complete mechanical obstruction in the intestine. Congenital aganglionic megacolon, also called Hirschsprung disease, is characterized by persistent constipation resulting from partial or complete intestinal obstruction of mechanical origin. A narrowing of the lumen of the pylorus is associated with pyloric stenosis in young infants. The telescoping of the bowel is intussusception. The relaxed sphincter in the lower portion of the esophagus is related to gastrointestinal reflux disorder.

The nurse is preparing teaching materials for a family whose child is prescribed somatropin for a growth hormone deficiency. What should the nurse instruct the parents about the administration of this medication? Hip or knee pain is an expected adverse effect of this medication. This medication must be given by injection. This medication must be given in the morning before school. This medication does not interact with any other types of medication.

This medication must be given by injection. Somatropin is administered by injection. It is best given at the hour of sleep because that is when growth hormone is released. Hip or knee pain could indicate a slipped capital epiphysis and should be reported to the health care provider. The nurse should urge the parents to inform all health care providers that the child is receiving this medication to avoid medication interactions.

A patient with suspected esophageal varices is scheduled for an upper endoscopy with moderate sedation. After the procedure is performed, how long should the nurse withhold food and fluids? For 6 hours after the procedure Until the gag reflex returns Until the patient expresses thirst For 2 hours after the last dose of medication is given

Until the gag reflex returns Explanation: After the endoscopic examination, fluids are not given until the patient's gag reflex returns.

An infant on the pediatric floor has diabetes insipidus. Which assessment data are important for the nurse to monitor while the infant is on strict fluid precautions? Oral intake Temperature and heart rate Urine output Color of mucous membranes

Urine output An infant with diabetes insipidus has a decrease in antidiuretic hormone. Strict fluid precautions will not alter urine formation. This assessment is important because the infant will be at great risk for dehydration and electrolyte imbalance. It is part of a basic assessment to monitor heart rate, temperature, skin turgor, and mucous membranes. These are important but may not indicate the infant's overall health. On fluid restriction, oral intake will be specified.

Which of the following is the most effective strategy to prevent hepatitis B infection? Barrier protection during intercourse Vaccine Covering open sores Avoid sharing toothbrushes

Vaccine The most effective strategy to prevent hepatitis B infection is through vaccination. Recommendations to prevent transmission of hepatitis B include vaccination of sexual contacts of individuals with chronic hepatitis, use of barrier protection during sexual intercourse, avoidance of sharing toothbrushes, razors with others, and covering open sores or skin lesions.

Which medication is used to decrease portal pressure, halting bleeding of esophageal varices? Nitroglycerin Spironolactone Cimetidine Vasopressin

Vasopressin Explanation: Vasopressin may be the initial therapy for esophageal varices because it constricts the splanchnic arterial bed and decreases portal hypertension. Nitroglycerin has been used to prevent the side effects of vasopressin. Spironolactone and cimetidine do not decrease portal hypertension.

A client with carcinoma of the head of the pancreas is scheduled for surgery. Which of the following should a nurse administer to the client before surgery? Vitamin K Vitamin B Potassium Oral bile acids

Vitamin K Explanation: Clients with carcinoma of the head of the pancreas typically require vitamin K before surgery to correct a prothrombin deficiency. Potassium would be given only if the client's serum potassium levels were low. Oral bile acids are not prescribed for a client with carcinoma of the head of the pancreas; they are given to dissolve gallstones. Vitamin B has no implications in the surgery.

The nurse is administering Cephulac (lactulose) to decrease the ammonia level in a patient who has hepatic encephalopathy. What should the nurse carefully monitor for that may indicate a medication overdose? Asterixis Ringing in the ears Watery diarrhea Vomiting

Watery diarrhea ??

Which of the following is a term used to describe a soft tissue injury produced by a blunt force? Strain Contusion Sprain Hematoma

Which term refers to the failure of fragments of a fractured bone to heal together? Nonunion Subluxation Dislocation Malunion

The nurse is helping to set up Buck's traction on an orthopedic client. How often should the nurse assess circulation to the affected leg? Within 30 minutes, then every 8 hours Within 30 minutes, then every shift Within 30 minutes, then every 1 to 2 hours Within 30 minutes, then every 4 hours

Within 30 minutes, then every 1 to 2 hours Explanation: After skin traction is applied, the nurse assesses circulation of the foot or hand within 15 to 30 minutes and then every 1 to 2 hours.

In a child with diabetes insipidus, which characteristic would most likely be present in the child's health history? vomiting early in the morning, headache, and decreased thirst gradual onset of personality changes, lethargy, and blurred vision abrupt onset of polyuria, nocturia, and polydipsia delayed closure of the fontanels (fontanelles), coarse hair, and hypoglycemia in the morning

abrupt onset of polyuria, nocturia, and polydipsia Diabetes insipidus is characterized by deficient secretion of antidiuretic hormone leading to diuresis. Most children with this disorder experience an abrupt onset of symptoms, including polyuria, nocturia, and polydipsia. The other choices reflect symptoms of pituitary hyperfunction.

A client has just been diagnosed with hepatitis A. On assessment, the nurse expects to note: abdominal ascites. severe abdominal pain radiating to the shoulder. anorexia, nausea, and vomiting. eructation and constipation.

anorexia, nausea, and vomiting. Early hallmark signs and symptoms of hepatitis A include anorexia, nausea, vomiting, fatigue, and weakness. Abdominal pain may occur but doesn't radiate to the shoulder. Eructation and constipation are common in gallbladder disease, not hepatitis A. Abdominal ascites is a sign of advanced hepatic disease, not an early sign of hepatitis A.

The nurse is reinforcing dietary teaching with the caregiver of a child diagnosed with celiac syndrome. Which foods would be permitted in the diet of the child with celiac syndrome? Select all that apply. rye bread skim milk wheat bread applesauce bananas

applesauce bananas skim milk The child is usually started on a gluten-free, low-fat diet.

A child with a history of diabetes insipidus is admitted with polyuria, polydipsia, and mental confusion. Which is the priority intervention for this child? weigh the client measure urine output check vital signs encourage increased fluid intake

check vital signs Explanation: Central diabetes insipidus is a disorder of the posterior pituitary. The fluid status of the child can be assessed first by assessing the vital signs. The large amounts of fluid loss can cause fluid and electrolyte imbalance that should be corrected. Urine output is important but not the priority. Encouraging fluids will not correct the problem, and weighing the client is not necessary at this time. Diabetes insipidus is managed by decreasing the protein and sodium in the diet and daily replacement of the antidiuretic hormone.

A client is admitted for suspected GI disease. Assessment data reveal muscle wasting, a decrease in chest and axillary hair, and increased bleeding tendency. The nurse suspects the client has: cholelithiasis. appendicitis. peptic ulcer disease. cirrhosis.

cirrhosis. Explanation: Muscle wasting, a decrease in chest and axillary hair, and increased bleeding tendencies are all symptoms of cirrhosis. The client may also have mild fever, edema, abdominal pain, and an enlarged liver. Clients with peptic ulcer disease complain of a dull, gnawing epigastric pain that's relieved by eating. Appendicitis is characterized by a periumbilical pain that moves to the right lower quadrant and rebound tenderness. Cholelithiasis is characterized by severe abdominal pain that presents several hours after a large meal.

Which is not one of the general nursing measures employed when caring for the client with a fracture? administering analgesics providing comfort measures cranial nerve assessment assisting with ADLs

cranial nerve assessment Explanation: Cranial nerve assessment would only be carried out for head-related injuries or diseases. General nursing measures include administering analgesics, providing comfort measures, assisting with ADLs, preventing constipation, promoting physical mobility, preventing infection, maintaining skin integrity, and preparing client for self-care.

A nurse is to see a child. Assessment reveals the chief complaints of urinating "a lot" and being "really thirsty." The nurse interprets these symptoms as being associated with which condition? syndrome of inappropriate antidiuretic hormone secretion precocious puberty hypopituitarism diabetes insipidus

diabetes insipidus The most common symptoms of central diabetes insipidus are polyuria (excessive urination) and polydipsia (excessive thirst). Children with diabetes insipidus typically excrete 4 to 15 L/day of urine despite the fluid intake. The onset of these symptoms is usually sudden and abrupt. Ask about repeated trips to the bathroom, nocturia, and enuresis. Other symptoms may include dehydration, fever, weight loss, increased irritability, vomiting, constipation, and, potentially, hypovolemic shock.

Insulin deficiency, in association with increased levels of counter-regulatory hormones and dehydration, is the primary cause of: ketonuria. ketone bodies. diabetic ketoacidosis. glucosuria.

diabetic ketoacidosis. Insulin deficiency, in association with increased levels of counter-regulatory hormones (glucagon, growth hormone, cortisol, catecholamines) and dehydration, is the primary cause of diabetic ketoacidosis (DKA), a life-threatening form of metabolic acidosis that is a frequent complication of diabetes. Liver converts triglycerides (lipolysis) to fatty acids, which in turn change to ketone bodies. The accumulation and excretion of ketone bodies by the kidneys is called ketonuria. Glucosuria is glucose that is spilled into the urine.

A client who has fallen and injured a hip cannot place weight on the leg and is in significant pain. After radiographs indicate intact but malpositioned bones, what would the physician diagnose? dislocation fracture strain sprain

dislocation Explanation: In joint dislocation, radiographic films show intact yet malpositioned bones. Arthrography or arthroscopy may reveal damage to other structures in the joint capsule. A strain is an injury to a muscle when it is stretched or pulled beyond its capacity. Sprains are injuries to the ligaments surrounding a joint. A fracture is a break in the continuity of a bone.

A client with acute pancreatitis has jaundice with diminished bowel sounds and a tender distended abdomen. Additionally, lab results indicate hypovolemia. What will the health care provider prescribe to treat the large amount of protein-rich fluid that has been released into the client's tissues and peritoneal cavity? Select all that apply. sodium albumin diuretics fentanyl dextrose solution

diuretics albumin Diuretics are given if circulating fluid is excessive. IV albumin may be given to pull fluid trapped in the peritoneum back into the circulation. Sodium would not be used to treat excessive fluid accumulation. Blood glucose levels can be elevated in clients with acute pancreatitis; therefore, glucose solutions would not be administered, nor would they be used to treat excessive fluid accumulation. Fentanyl is given if the client is experiencing pain, not to correct fluid imbalance.

The nurse is teaching a child with type 1 diabetes mellitus to administer insulin. The child is receiving a combination of short-acting and long-acting insulin. The nurse knows that the child has appropriately learned the technique when the child: administers the insulin into a doll at a 30-degree angle. administers the insulin intramuscularly into rotating sites. wipes off the needle with an alcohol swab. draws up the short-acting insulin into the syringe first.

draws up the short-acting insulin into the syringe first. Drawing up the short-acting insulin first prevents mixing a long-acting form into the vial of short-acting insulin. This maintains the short-acting insulin for an emergency. Insulin is given subcutaneously not intramuscularly. A SQ injection is administered at a 90-degree angle if the person can grasp 2 in (5 cm) of skin. If only 1 in (2.5 cm) of skin can be grasped, then the injection should be given at a 45 degree angle.

Which nursing objective is most important when working with neonates who are suspected of having congenital hypothyroidism? early identification encouraging fluid intake promoting bonding allowing rooming in

early identification

A child is admitted to the pediatric medical unit with the diagnosis of syndrome of inappropriate antidiuretic hormone (SIADH). The child experiences the typical signs and symptoms of this disorder. Which concern will the nurse include in care planning? delayed growth and development risk excess fluid volume risk altered nutrition risk noncompliance because of difficulty coping

excess fluid volume risk Syndrome of inappropriate antidiuretic hormone (SIADH) occurs when antidiuretic hormone (ADH; vasopressin) is secreted in the presence of low serum osmolality because the feedback mechanism that regulates ADH does not function properly. ADH continues to be released, and this leads to water retention, decreased serum sodium due to hemodilution, and extracellular fluid volume expansion; thus, the priority concern for care planning is the risk for excess fluid volume from edema.

A pediatric client has just been diagnosed with diabetes insipidus. What is the primary consideration for this client? fluid replacement polydipsia headache weight loss

fluid replacement Children with diabetes insipidus lose tremendous amounts of fluid, so fluid replacement is the priority consideration for this client. Excessive fluid loss can lead to seizures and death. Headache and polydipsia can be relieved with fluid replacement. Children will requirement a nutritional consultation for weight loss, but it is not the main consideration.

The type of fracture described as having one side of the bone broken and the other side bent would be: transverse. oblique. spiral. greenstick.

greenstick. Explanation: A greenstick fracture is the type of fracture described as having one side of the bone broken and the other side bent. An oblique fracture occurs at an angle across the bone. A spiral fracture is a fracture that twists around the shaft of the bone. A transverse fracture is a fracture that is straight across the bone.

The nurse knows that disorders of the pituitary gland depend on the location of the physiologic abnormality. In caring for a child that has issues with the anterior pituitary, the nurse knows that this child has issues with which hormone? antidiuretic hormone growth hormone vasopressin oxytocin

growth hormone Disorders of the pituitary gland depend on the location of the physiologic abnormality. The anterior pituitary, or adenohypophysis, is made up of endocrine glandular tissue and secretes growth hormone (GH), adrenocorticotropic hormone (ACTH), TSH, follicle-stimulating hormone (FSH), luteinizing hormone (LH), and prolactin. The posterior lobe is called the neurohypophysis because it is formed of neural tissue. It secretes antidiuretic hormone (ADH; vasopressin) and oxytocin. Usually, several target organs are affected when there is a disorder of the pituitary gland, especially the adenohypophysis.

The nurse knows that disorders of the pituitary gland depend on the location of the physiologic abnormality. In caring for a child that has issues with the anterior pituitary, the nurse knows that this child has issues with which hormone? antidiuretic hormone vasopressin growth hormone oxytocin

growth hormone Disorders of the pituitary gland depend on the location of the physiologic abnormality. The anterior pituitary, or adenohypophysis, is made up of endocrine glandular tissue and secretes growth hormone (GH), adrenocorticotropic hormone (ACTH), TSH, follicle-stimulating hormone (FSH), luteinizing hormone (LH), and prolactin. The posterior lobe is called the neurohypophysis because it is formed of neural tissue. It secretes antidiuretic hormone (ADH; vasopressin) and oxytocin. Usually, several target organs are affected when there is a disorder of the pituitary gland, especially the adenohypophysis.

The nurse is conducting a physical examination of an infant with suspected pyloric stenosis. Which finding indicates pyloric stenosis? perianal fissures and skin tags abdominal pain and irritability hard, moveable "olive-like mass" in the upper right quadrant sausage-shaped mass in the upper mid abdomen

hard, moveable "olive-like mass" in the upper right quadrant A hard, moveable "olive-like mass" in the right upper quadrant is the hypertrophied pylorus. A sausage-shaped mass in the upper mid abdomen is the hallmark of intussusception. Perianal fissures and skin tags are typical with Crohn disease. Abdominal pain and irritability is common with pyloric stenosis but are seen with many other conditions.

A nurse in the surgical intensive care unit (ICU) just received a client from recovery following a Whipple procedure. For which complication(s) should the nurse monitor while caring for the client? Select all that apply. pancreatic abscess formation hemorrhage shock fluid overload overnutrition

hemorrhage shock ??

The parent reports that the health care provider said that the infant had a hernia but cannot remember which type. When recalling what the health care provider said, the parent said that a surgeon will repair it soon and there is no problem with the testes. Which hernia type is anticipated? diaphragmatic hernia hiatal hernia inguinal hernia umbilical hernia

inguinal hernia An inguinal hernia occurs primarily in males and allows the intestine to slip into the inguinal canal, resulting in swelling. If the intestine becomes trapped and circulation is impaired, surgery is indicated within a short period of time. The diaphragmatic hernia has implications with the respiratory system. An umbilical hernia typically spontaneously closes by age 3. A hiatal hernia produces digestive issues.

The nurse is interpreting the negative feedback system that controls endocrine function. What secretion will the nurse correlate as decreasing while blood glucose levels decrease glycogen insulin adrenocorticotropic hormone glucagon

insulin Explanation: Feedback is seen in endocrine systems that regulate concentrations of blood components such as glucose. Glucose from the ingested lactose or sucrose is absorbed in the intestine and the level of glucose in blood rises. Elevation of blood glucose concentration stimulates endocrine cells in the pancreas to release insulin. Insulin has the major effect of facilitating entry of glucose into many cells of the body; as a result, blood glucose levels fall. When the level of blood glucose falls sufficiently, the stimulus for insulin release disappears and insulin is no longer secreted.

A client with a lengthy history of alcohol addiction is being seen for jaundice. What does the appearance of jaundice most likely indicate? glucose underproduction gallbladder disease bile overproduction liver disorder

liver disorder Jaundice is a sign of disease, but it is not itself a unique disease. Jaundice accompanies many diseases that directly or indirectly affect the liver and is probably the most common sign of a liver disorder.

What is the recommended dietary treatment for a client with chronic cholecystitis? high-fiber diet low-protein diet low-fat diet low-residue diet

low-fat diet Explanation: The bile secreted from the gallbladder helps the body absorb and break down dietary fats. If the gallbladder is not functioning properly, then it will not secrete enough bile to help digest the dietary fat. This can lead to further complications; therefore, a diet low in fat can be used to prevent complications.

The nurse teaches a parent to differentiate between regurgitation and vomiting in the infant. The parent correctly states which characteristic of regurgitation? Select all that apply. occurs with feeding no appearance of distress followed by dry retching forceful expulsion of stomach contents timing unrelated to feeding

occurs with feeding no appearance of distress Regurgitation occurs with feeding; the infant does not exhibit signs of distress. Forceful expulsion of stomach contents that is followed by dry retching unrelated to feeding are characteristics of vomiting.

The nurse is caring for a child recently diagnosed with hypoparathyroidism disorder. Which medication would the nurse expect to be ordered? intravenous diuretic therapy oral corticosteroids oral potassium oral calcium

oral calcium Medical management of hypoparathyroidism includes intravenous calcium gluconate for acute or severe tetany, then intramuscular or oral calcium as prescribed. IV diuretics are used in the treatment of hyperparathyroidism. Oral corticosteroids and oral potassium are not used in the treatment of hypoparathyroidism.

A parent brings a 10-year-old child to the emergency room with reports of abdominal pain. The nurse performing a physical assessment notes the following symptoms: upper right quadrant pain that radiates to the back; fever; nausea; and abdominal distention. Which disease would the nurse suspect? Crohn disease appendicitis pancreatitis ulcerative colitis

pancreatitis The child admitted with the suspicion of pancreatitis typically reports acute onset of persistent abdominal pain. It can be mid-epigastric or periumbilical with radiation to the back or the chest. Nausea and vomiting, fever, tachycardia, hypotension, and jaundice may be present. Abdominal signs such as abdominal distention, decreased bowel sounds, rebound tenderness, and guarding also may be noted. Appendicitis pain and tenderness would be localized to the right lower quadrant. Crohn disease is a chronic bowel disorder causing frequent, recurring diarrhea. Ulcerative colitis is a chronic bowel disease affecting the large intestine and the rectum.

Most of the liver's metabolic functions are performed by: parenchymal cells. canaliculi cells. Kupffer cells. islets of Langerhans.

parenchymal cells. Explanation: The parenchymal cells perform most of the liver's metabolic functions.

Which is the best way to control enzyme deficiencies? herbs and natural foods special diet restrictions and synthetic medical foods high-protein and low-sodium diet high-carbohydrate and low-fat diet

special diet restrictions and synthetic medical foods Explanation: The goal of dietary restriction, the primary treatment modality for inborn errors of metabolism, is to control the substrate accumulation by reducing or eliminating carbohydrates, proteins, or both. Special diet restrictions and synthetic medical foods are the two most successful methods of controlling enzyme deficiencies.

The caregiver of a child diagnosed with celiac disease tells the nurse that the child has large amounts of bulky stools and what looks like fat in the stools. The clinical manifestation this caregiver is describing is: projectile stools. currant jelly stools. steatorrhea. severe diarrhea.

steatorrhea. Celiac disease is an immunologic response to gluten, which causes damage to the small intestine. Steatorrhea (fatty stools) is a classic symptom of celiac disease. Symptoms also include abdominal distention or bloating, constipation, and nutritional deficiencies. Currant jelly stools are a sign of intussusception. Projectile vomiting is a sign of pyloric stenosis. Severe diarrhea could be caused by a bacteria or virus. Projectile stools represent severe diarrhea.

A client has been diagnosed with a muscle strain. What does the physician mean by the term "strain"? injuries to ligaments surrounding a joint subluxation of a joint injury resulting from a blow or blunt trauma stretched or pulled beyond its capacity

stretched or pulled beyond its capacity Explanation: A strain is an injury to a muscle when it is stretched or pulled beyond its capacity.

A client with acute liver failure exhibits confusion, a declining level of consciousness, and slowed respirations. The nurse finds him very difficult to arouse. The diagnostic information which best explains the client's behavior is: subnormal clotting factors and platelet count. subnormal serum glucose and elevated serum ammonia levels. elevated liver enzymes and low serum protein level. elevated blood urea nitrogen and creatinine levels and hyperglycemia.

subnormal serum glucose and elevated serum ammonia levels. Explanation: In acute liver failure, serum ammonia levels increase because the liver can't adequately detoxify the ammonia produced in the GI tract. In addition, serum glucose levels decline because the liver isn't capable of releasing stored glucose. Elevated serum ammonia and subnormal serum glucose levels depress the level of a client's consciousness. Elevated liver enzymes, low serum protein level, subnormal clotting factors and platelet count, elevated blood urea nitrogen and creatine levels, and hyperglycemia aren't as directly related to the client's level of consciousness.

A 12-year-old client arrives at the emergency room experiencing nausea, vomiting, headache, and seizures. The client is diagnosed with bacterial meningitis. Other findings include a decrease in urine production, hyponatremia, and water intoxication. Which pituitary gland disorder is most associated with these symptoms? hypersecretion of growth hormone diabetes insipidus (DI) syndrome of inappropriate antidiuretic hormone (SIADH) hyposecretion of somatotropin

syndrome of inappropriate antidiuretic hormone (SIADH) Syndrome of inappropriate antidiuretic hormone (SIADH) is a rare condition in which there is overproduction of antidiuretic hormone by the posterior pituitary gland. This results in a decrease in urine production and water intoxication. As sodium levels fall in proportion to water, the child develops hyponatremia or a lowered sodium plasma level. It can be caused by central nervous system infections such as bacterial meningitis. As the hyponatremia grows more severe, coma or seizures occur from brain edema. Diabetes insipidus is characterized by polyuria, not decreased urine production. Hyposecretion of somatotropin (growth hormone) results in undergrowth; hypersecretion results in overgrowth.

The nurse working with the child diagnosed with type 2 diabetes recognizes the disorder can be managed by: taking oral hypoglycemic agents. increasing carbohydrates in the diet, especially in the evening. decreasing amounts of daily insulin. conserving energy with rest periods during the day.

taking oral hypoglycemic agents. Oral hypoglycemic agents, such as metformin, are often effective for controlling blood glucose levels in children diagnosed with type 2 diabetes. Insulin may be used for a child with type 2 diabetes if oral hypoglycemic agents alone are not effective, but "decreasing" the daily insulin would not help treat this disorder. Lifestyle changes such as increased exercise (not conserving energy by resting during the day), and limiting large amounts of carbohydrates are important aspects of treatment for the child.


Kaugnay na mga set ng pag-aaral

[Toán] Thơ về công thức lượng giác rất dễ nhớ

View Set

Біологічне окиснення

View Set

6th Grade Science: Chapter 7: Atoms & Molecules

View Set

Financial Planning Glossary- Annuities CE Course

View Set

The Development of Feudalism in Western Europe

View Set

Unit 2: Energy and Energy Resources

View Set

Obecná psychologie LS 2021 TEST

View Set

Lecture 18: Esophageal Problems, Stomach Disorders, and Biliary System Problems

View Set

CH 6 (Deductive Reasoning: Propositional Logic)

View Set

الفصل ٣ - القضايا الأخلاقية والمهنية في ممارسة الإرشاد الجمعي

View Set